Sobre el problema 3 del selectivo final

Versión para impresión
Voy a presentar en este post la solución al problema 3 del selectivo final para la preselección Tamaulipas OMM 2014. Añado una solución alternativa con un algoritmo para resolver la ecuación de Pell. (De paso, con esta solución alternativa, puede verse el poder del procedimiento --sin entrar en detalles de por qué funciona.)
 

El problema y la solución de Germán

La solución de Germán procede mediante inferencias de divisibilidad. En ese sentido es una solución muy básica. El problema es el siguiente.
 
Encuentra el menor entero positivo $n\gt1$ tal que el promedio de $1^2,2^2,3^2,\ldots,n^2$ es un cuadrado perfecto.
Solución
 
Aplicando la fórmula de la suma de los primeros cuadrados perfectos hasta n y simplificando se llega a la ecuación diofantina
$$(n+1)(2n+1)=6k^2$$
Puesto que $2n+1$ siempre es impar entonces $n+1$ debe ser par. Digamos que $n=2i-1$. La diofantina se simplifica entonces a 
$$i(4i-1)=3k^2$$
 
Es fácil ver en esta ecuación que los factores del lado izquierdo son coprimos (cualquier divisor común divide también a la diferencia de $4i$ y $4i-1$, es decir, divide a -1). Tenemos entonces dos casos: $i=3m$ o $i=3m+1$.
 
Caso 1: i=3m
 
Si i=3m entonces la ecuación se simplifica a 
$$m(12m-1)=k^2$$
 
Puesto que los dos factores son coprimos, se sigue que ambos deben ser cuadrados perfectos. Pero esto es imposible pues 12m-1 deja 2 de residuo en la división entre 3. Así que este caso no es posible.
 
Caso 2: i=3m+1
 
Si i=3m+1 la ecuación se simplifica a 
$$(3m+1)(4m+1)=k^2$$
 
Puesto que cualquier divisor común debe dividir a la diferencia de 4 veces el primer factor menos 3 veces el segundo se sigue que ambos factores son coprimos. De aquí que ambos deben ser cuadrados perfectos.
 
Si $3m+1=y^2$ entonces $3m=(y-1)(y+1)$.
Si $4m+1=z^2$ entonces $4m=(z-1)(z+1)$ y se ve que ambos factores del lado derecho deben ser pares. De aquí que $m=r(r+1)$.
 
Pero entonces $(y-1)(y+1)=3r(r+1)$. De aquí es fácil concluir que el lado izquierdo es par. Pero entonces es múltiplo de 4. Y como sus factores son pares consecutivos uno de ellos es múltiplo de 4. En resumen, $r(r+1)=8s$. Y como buscamos el mínimo, basta con elegir r=7. 
 
Pero si r=7 entonces m=56, i=3(56)+1 y n=6(56)+2-1. Por tanto, el número buscado es n=337. (Es fácil verificar que, en ese caso, la media de la suma de cuadrados hasta 337 es el cuadrado perfecto $13^2\cdot 15^2$. (Solución reconstruida a partir de la de Germán Puga)
 

Solución alternativa y cómo la descubrí

Como si fuera un acertijo, después del selectivo final me pasé varios días buscando la solución del problema 3. No puede estar muy difícil --me dije-- dado que es una ecuación diofantina. (Aunque hay que decir que no lineal.) Pero no encontraba el camino hacia su solución. 
 
En el entrenamiento siguiente yo tuve que atender a la selección el viernes en la tarde --pues Hector Flores empezaría el sábado en la mañana. Así que estuve presente cuando los seleccionados le solicitaron a Germán Puga explicara su solución al problema 3. (También explicó la solución del 2.) 
 
Debido a que la solución de Germán es cristalina, es decir, fluye lógicamente de un paso a otro, no tomé ningún apunte. Pero cuando intenté recostruirla en mi casa me faltaba algo. (Como se puede ver en la solución ya presentada, el detalle fino se encuentra al final: la resolución de dos ecuaciones simultáneas mediante inferencias de divisibilidad. )
 
Después de lograr reconstruirla --gracias a los apuntes que tomó Jesús Anaya-- consulté a Jesús Rodríguez Viorato sobre una solución alternativa. Me dijo que él lo resolvió con la ecuación de Pell. 
 
Y, bueno, voy a tratar en lo que sigue de explicar cómo se usa la ecuación de Pell para resolver este problema. Esperando que el caso particular deje alguna enseñanza a los lectores de MaTeTaM sobre el caso general.
 
Voy a abordar la ecuación de Pell desde la perspectiva ingenieril. Es decir, voy a dar el procedimiento de solución y a mostrar que sí funciona ejemplificando con el problema que nos ocupa. (Y si alguien desea la demostración de por qué funciona puede investigar en la Web.)
 

La ecuación de Pell

La ecuación de Pell es una ecuación diofantina (se pide resolverla en números enteros) y es legendaria. (Google arroja 7480 resultados.) Tiene la siguiente forma: $Dx^2+1=y^2$, donde D es un entero positivo pero no cuadrado perfecto. Se pide resolverla para $x,y$ enteros. (Se puede demostrar que si D no es cuadrado perfecto la ecuación de Pell admite infinitas soluciones.)
 
Antes de presentar la solución alternativa permítaseme presentar el algoritmo con el que la resolví (gracias a la sugerencia de Jesús Rodríguez Viorato). Cabe destacar que éste es uno de los varios métodos que permiten resolver la ecuación de Pell y, debido a esa abundancia, el que presento aquí queda bastante oculto en las páginas de Internet dedicadas a Pell. (La moraleja es que en la Web está todo... ¡pero hay que saber descubrirlo!) El algoritmo es el siguiente:
 
 
1. Encontrar la solución mínima por inspección. Digamos que sea $(x_1,y_1)$.
2. Encontrar las restantes mediante la ecuación $(y_1+\sqrt{D})^n=y_n+\sqrt{D}x_n$
 
Veamos ahora su funcionamiento resolviendo la ecuación $8x^2+1=y^2$. Es relativamente fácil obtener la solución mínima $(x_1,y_1)=(1,3)$ por inspección. Entonces, según el procedimiento, $(x_2,y_2)$ se obtiene con la ecuación $(3+\sqrt{8})^2=y_2+\sqrt{8}x_2.$ Véase:
 
$$9+6\sqrt{8}+8=y_2+\sqrt{8}x_2$$
$$17+6\sqrt{8}=y_2+\sqrt{8}x_2$$
 
De aquí que $x_2=6,y_2=17$
 

Solución alternativa con Pell

Como se dijo antes, el enunciado del problema 3 conduce a la ecuación diofantina
$$(n+1)(2n+1)=6k^2$$
Y mediante manipulaciones algebraicas básicas (multiplicando por 2 y completando el trinomio cuadrado perfecto en el lado izquierdo) se llega a la ecuación
$$(4n+3)^2-1=48k^2$$ 
Y mediante el cambio de variable $y=4n+3, x=k$ se logra la ecuación de Pell 
$$48x^2+1= y^2$$
Es fácil ver que la solución mínima es $(x_1,y_1)=(1,7)$. Para el paso 2 se resuelve la ecuación
$$(7+\sqrt{48})^2=y_2+\sqrt{48}x_2$$
Y como $(7+\sqrt{48})^2=97+14\sqrt{48}$, la siguiente solución es $(x_2,y_2)=(14,97)$.
 
Es fácil ver que, para esta solución (después de regresar a la variable original), n es fraccionario. Por tanto no funciona para nuestro problema. Veamos la siguiente (las cuentas para el lector):
 
$$(7+\sqrt{48})^3=1351+195\sqrt{48}$$
Así que la segunda solución es $(195,1351)$. Pero si $y=1351=4n+3$ entonces $n=337$. Acabamos.
 
Los saluda
jmd